Indução - Putnam 2009
2 participantes
PiR2 :: Questões Especiais :: Olimpíadas :: Matemática
Página 1 de 1
Indução - Putnam 2009
(Putnam 2009) Para n inteiro positivo, mostre que
[latex]\sum_{k=1}^{n}\sqrt{k} < \frac{4n + 3}{6}.\sqrt{n}[/latex]
Não queria a resolução diretamente. Se puderem deixar umas dicas/ideias antes da resolução (ou a resolução como spoiler), ajudaria bastante.
Não queria a resolução diretamente. Se puderem deixar umas dicas/ideias antes da resolução (ou a resolução como spoiler), ajudaria bastante.
gabriel_balbao- Padawan
- Mensagens : 92
Data de inscrição : 03/02/2021
Idade : 21
Localização : Ribeirão Preto
Re: Indução - Putnam 2009
Olá Gabriel;
Defina-se [latex]S_n = \sum_{k=1}^{n}\sqrt{k} [/latex]
Então [latex] S_{n+1} = S_n + \sqrt{n+1}[/latex]
Suponha que para algum n tenha-se [latex] S_{n} < \frac{4n + 3}{6}\sqrt n[/latex]. Nessas condições:
[latex]\scriptsize{ S_{n+1} - \sqrt{n+1} = S_n \text{ }, \text{ } \text{ } S_{n} < \frac{4n + 3}{6}\sqrt n \implies (S_{n+1} - \sqrt{n+1}) < \frac{4n + 3}{6}\sqrt{n} \implies S_{n+1} < (\sqrt{n+1} + \frac{4n + 3}{6}\sqrt{n})}[/latex]
Deseja-se provar que:
[latex]S_{n+1} < (\frac{(4(n+1) +3)}{6}\cdot \sqrt{n+1})[/latex]
Hipótese inicial:
Verificação da hipótese:
Bons estudos
Defina-se [latex]S_n = \sum_{k=1}^{n}\sqrt{k} [/latex]
Então [latex] S_{n+1} = S_n + \sqrt{n+1}[/latex]
Suponha que para algum n tenha-se [latex] S_{n} < \frac{4n + 3}{6}\sqrt n[/latex]. Nessas condições:
[latex]\scriptsize{ S_{n+1} - \sqrt{n+1} = S_n \text{ }, \text{ } \text{ } S_{n} < \frac{4n + 3}{6}\sqrt n \implies (S_{n+1} - \sqrt{n+1}) < \frac{4n + 3}{6}\sqrt{n} \implies S_{n+1} < (\sqrt{n+1} + \frac{4n + 3}{6}\sqrt{n})}[/latex]
Deseja-se provar que:
[latex]S_{n+1} < (\frac{(4(n+1) +3)}{6}\cdot \sqrt{n+1})[/latex]
Hipótese inicial:
- Spoiler:
A tentativa mais simples seria verificar para quais n pode-se armar a inequação:
[latex] \sqrt{n+1} + \frac{4n + 3}{6}\sqrt{n} < \frac{(4(n+1) +3)}{6}\cdot \sqrt{n+1} [/latex]
Pois assim você pode usar a transitividade das desigualdades para armar a senteça (I):
[latex]\scriptsize{ S_{n+1} <(\sqrt{n+1} + \frac{4n + 3}{6}\sqrt{n}) \text{ }, \text{ } \text{ } (\sqrt{n+1} + \frac{4n + 3}{6}\sqrt{n}) <( \frac{[4(n+1) +3]}{6}\cdot \sqrt{n+1}) \implies S_{n+1} < (\frac{(4(n+1) +3)}{6}\cdot \sqrt{n+1})}[/latex]
Verificação da hipótese:
- Spoiler:
[latex] \scriptsize{(\sqrt{n+1} + \frac{4n + 3}{6}\sqrt{n}) < (\frac{[4(n+1) +3]}{6}\cdot \sqrt{n+1}) \implies \frac{4n + 3}{6}\sqrt{n} < (\frac{[4(n+1) +3]}{6}\cdot \sqrt{n+1} - \sqrt{n+1}) \implies}[/latex]
[latex]\scriptsize{\frac{4n + 3}{6}\sqrt{n} < \frac{[4n +1]}{6}\cdot \sqrt{n+1} \implies (4n+3)\sqrt n < (4n+1)\sqrt{n+1} \implies{ ((4n+3)\sqrt n )^2} <{((4n+1)\sqrt{n+1} )^2} \implies}[/latex]
[latex](16n^3 + 24n^2 + 9n )< (16n^3 + 8n^2 +n + 16 n^2 + 8n +1) \implies 0 < 1[/latex]
Ou seja, se [latex] S_{n} < \frac{4n + 3}{6}\sqrt n[/latex] então [latex] \sqrt{n+1} + \frac{4n + 3}{6}\sqrt{n} < \frac{(4(n+1) +3)}{6}\cdot \sqrt{n+1} [/latex] é verdade. Assim (I) está comprovada.
Para finalizar a indução basta provar o caso inicial n=1:
[latex]S_1 < \frac{4\cdot 1 + 3}{6} \Leftrightarrow 1 < \frac{7}{6}[/latex]
Bons estudos
joaoZacharias- Recebeu o sabre de luz
- Mensagens : 134
Data de inscrição : 18/03/2020
Localização : Campinas - SP, BR
qedpetrich gosta desta mensagem
Tópicos semelhantes
» (Putnam) matrizes
» (Putnam-55) Números complexos.
» (Putnam/1989) Números complexos IV
» Putnam 1967 - Geometria com Complexos
» PUTNAM 1992 - MUITO DIFÍCIL
» (Putnam-55) Números complexos.
» (Putnam/1989) Números complexos IV
» Putnam 1967 - Geometria com Complexos
» PUTNAM 1992 - MUITO DIFÍCIL
PiR2 :: Questões Especiais :: Olimpíadas :: Matemática
Página 1 de 1
Permissões neste sub-fórum
Não podes responder a tópicos